Last visit was: 25 Apr 2024, 07:14 It is currently 25 Apr 2024, 07:14

Close
GMAT Club Daily Prep
Thank you for using the timer - this advanced tool can estimate your performance and suggest more practice questions. We have subscribed you to Daily Prep Questions via email.

Customized
for You

we will pick new questions that match your level based on your Timer History

Track
Your Progress

every week, we’ll send you an estimated GMAT score based on your performance

Practice
Pays

we will pick new questions that match your level based on your Timer History
Not interested in getting valuable practice questions and articles delivered to your email? No problem, unsubscribe here.
Close
Request Expert Reply
Confirm Cancel
SORT BY:
Date
Math Expert
Joined: 02 Sep 2009
Posts: 92913
Own Kudos [?]: 618949 [66]
Given Kudos: 81595
Send PM
Most Helpful Reply
Math Expert
Joined: 02 Sep 2009
Posts: 92913
Own Kudos [?]: 618949 [33]
Given Kudos: 81595
Send PM
Math Expert
Joined: 02 Sep 2009
Posts: 92913
Own Kudos [?]: 618949 [17]
Given Kudos: 81595
Send PM
General Discussion
avatar
Intern
Intern
Joined: 14 Jul 2013
Posts: 34
Own Kudos [?]: 14 [0]
Given Kudos: 43
Location: United States
GMAT 1: 650 Q47 V32
Send PM
Re: D01-10 [#permalink]
Hi Bunuel,
I appreciate your answer, but the method looks bit clumsy to me.
I solved it in simpler way, please guide me whether my approach is correct.

1) 1/x > -1. It means 1 > -x
That is x > -1 .
So we are not sure that if the x is greater than 1. So it is insufficient.
2) 1/x^5 > 1/x^3. It means 1/x^2 > 1 .
That is x^2 < 1.
If x^2 is less than 1, that definitely implies x is less than 1. So it is sufficient.
User avatar
Manager
Manager
Joined: 04 Oct 2013
Posts: 130
Own Kudos [?]: 298 [9]
Given Kudos: 55
Location: India
GMAT Date: 05-23-2015
GPA: 3.45
Send PM
Re: D01-10 [#permalink]
7
Kudos
2
Bookmarks
Is x greater than 1?


(1) 1/x > -1

(2) (1/x)^5 >(1/x)^3



Alternate solution (Substituting numbers)
Attachments

Is x greater than 1.png
Is x greater than 1.png [ 29.39 KiB | Viewed 103108 times ]

Intern
Intern
Joined: 02 Jul 2018
Posts: 19
Own Kudos [?]: 23 [0]
Given Kudos: 27
Send PM
Re: D01-10 [#permalink]
Hi Bunuel, VeritasKarishma or anyone: I tried and tried and tried but I am not able to comprehend how is B Sufficient. I have tried to solve it via Wavy Curve Method, can you please help me understand what I am doing wrong
Attachments

File comment: Wavy Method
IMG_20190704_002841__01.jpg
IMG_20190704_002841__01.jpg [ 2.3 MiB | Viewed 12877 times ]

Tutor
Joined: 16 Oct 2010
Posts: 14822
Own Kudos [?]: 64911 [1]
Given Kudos: 426
Location: Pune, India
Send PM
Re: D01-10 [#permalink]
1
Kudos
Expert Reply
amanrsingh wrote:
Hi Bunuel, VeritasKarishma or anyone: I tried and tried and tried but I am not able to comprehend how is B Sufficient. I have tried to solve it via Wavy Curve Method, can you please help me understand what I am doing wrong


Your stmnt 2 analysis is not correct.

\(1/x^5 > 1/x^3\)

\(1/x^5 - 1/x^3 > 0\)

\((1 - x^2)/x^5 > 0\)

\((x^2 - 1)/x^5 < 0\) (Multiply both sides by -1)

\((x + 1)(x - 1)/x^5 < 0\) (Note that to use the wavy line method as discussed, we must have factors in the form (ax + b) or (ax - b), not (b - ax))

Transition points, -1, 0, 1

The expression is negative when 0 < x< 1 or x < -1

So in any case, x is less than 1.
Intern
Intern
Joined: 09 Oct 2019
Posts: 43
Own Kudos [?]: 4 [0]
Given Kudos: 65
Send PM
Re: D01-10 [#permalink]
Hi Bunuel,

In this question, we were not given the following info:

x is not equal to 0.

We solved the question without this. Please let me know if this was to inferred from the 2 statements given in the question. I solved the question with the approach same as yours but I had to assume that x is not equal to 0 here.

Does it make any difference whether that was given or not, as I feel this is quite an important piece of information?

Thanks.
Math Expert
Joined: 02 Sep 2009
Posts: 92913
Own Kudos [?]: 618949 [1]
Given Kudos: 81595
Send PM
Re: D01-10 [#permalink]
1
Kudos
Expert Reply
theeliteguy wrote:
Hi Bunuel,

In this question, we were not given the following info:

x is not equal to 0.

We solved the question without this. Please let me know if this was to inferred from the 2 statements given in the question. I solved the question with the approach same as yours but I had to assume that x is not equal to 0 here.

Does it make any difference whether that was given or not, as I feel this is quite an important piece of information?

Thanks.



From each statement we could deduce that x cannot be 0 because if x were 0, the statements would not be correct. For example, 1/0 > -1 would not be correct because 1/0 is undefined and cannot be greater than -1.
Director
Director
Joined: 28 Sep 2018
Posts: 734
Own Kudos [?]: 559 [0]
Given Kudos: 248
GMAT 1: 660 Q48 V33 (Online)
GMAT 2: 700 Q49 V37
Send PM
Re: D01-10 [#permalink]
Bunuel wrote:
Is \(x\) greater than 1?


(1) \(\frac{1}{x} \gt -1\)

(2) \(\frac{1}{x^5} \gt \frac{1}{x^3}\)


GMATBusters GMATinsight is 1/a < 1/b we can invert the two fractions and say a > b right? If that is true then for (1) why can't we say 1/x > -1 so x < 1/-1 i.e. x < -1 ?
GMAT Club Legend
GMAT Club Legend
Joined: 08 Jul 2010
Status:GMAT/GRE Tutor l Admission Consultant l On-Demand Course creator
Posts: 5960
Own Kudos [?]: 13387 [0]
Given Kudos: 124
Location: India
GMAT: QUANT+DI EXPERT
Schools: IIM (A) ISB '24
GMAT 1: 750 Q51 V41
WE:Education (Education)
Send PM
Re: D01-10 [#permalink]
Expert Reply
Hoozan wrote:
Bunuel wrote:
Is \(x\) greater than 1?


(1) \(\frac{1}{x} \gt -1\)

(2) \(\frac{1}{x^5} \gt \frac{1}{x^3}\)


GMATBusters GMATinsight is 1/a < 1/b we can invert the two fractions and say a > b right? If that is true then for (1) why can't we say 1/x > -1 so x < 1/-1 i.e. x < -1 ?


Hoozan

Rules of Cross multiplication in case of an Inequality
1) Cross multiplication of positive values doesn't affect the Inequality Sign
2) If a negative value shifts from one side to the other side then the inequality Sign FLips

Therefore,
1/a < 1/b will be a >b only if a and b are both positive (No change in sign) or both Negative (sign changes twice and remains the same)

Alternatively, You could understand it this way
If a*b is Positive then you can multiply a*b both sides of inequation without changing the inequality sign

That is why
1/x > -1 will become x < 1/-1 by multiplying -x both sides only when -x is Positive i.e. when x is Negative

I hope it clarifies your doubt. :)
Manager
Manager
Joined: 17 Jun 2020
Posts: 105
Own Kudos [?]: 8 [0]
Given Kudos: 314
Location: India
Schools: Simon '25
Send PM
Re: D01-10 [#permalink]
Bunuel wrote:
amitjathar wrote:
Hi Bunuel,
I appreciate your answer, but the method looks bit clumsy to me.
I solved it in simpler way, please guide me whether my approach is correct.

1) 1/x > -1. It means 1 > -x
That is x > -1 .
So we are not sure that if the x is greater than 1. So it is insufficient.
2) 1/x^5 > 1/x^3. It means 1/x^2 > 1 .
That is x^2 < 1.
If x^2 is less than 1, that definitely implies x is less than 1. So it is sufficient.


No, that's not correct.

You cannot multiply 1/x > -1 by x because you don't know the sign of x. If x is positive then yes, 1 > -x but if x is negative, then when multiplying by it you should flip the sign and you get 1 < -x.

Similarly you cannot reduce 1/x^5 > 1/x^3 by 1/x^3, because you don't know its sign.

Never multiply (or reduce) an inequality by a variable (or the expression with a variable) if you don't know its sign.

Other tips on Inequalities are here: https://gmatclub.com/forum/inequalities- ... 75001.html



Hi bunuel,
Can we not simply flip the inequality sign by taking reciprocal on both sides since both sides have odd powers (ie both shall have same signs) ?
so we'll get x^5 < x^3
Is this logic correct?
Math Expert
Joined: 02 Sep 2009
Posts: 92913
Own Kudos [?]: 618949 [1]
Given Kudos: 81595
Send PM
Re: D01-10 [#permalink]
1
Bookmarks
Expert Reply
SDW2 wrote:
Bunuel wrote:
amitjathar wrote:
Hi Bunuel,
I appreciate your answer, but the method looks bit clumsy to me.
I solved it in simpler way, please guide me whether my approach is correct.

1) 1/x > -1. It means 1 > -x
That is x > -1 .
So we are not sure that if the x is greater than 1. So it is insufficient.
2) 1/x^5 > 1/x^3. It means 1/x^2 > 1 .
That is x^2 < 1.
If x^2 is less than 1, that definitely implies x is less than 1. So it is sufficient.


No, that's not correct.

You cannot multiply 1/x > -1 by x because you don't know the sign of x. If x is positive then yes, 1 > -x but if x is negative, then when multiplying by it you should flip the sign and you get 1 < -x.

Similarly you cannot reduce 1/x^5 > 1/x^3 by 1/x^3, because you don't know its sign.

Never multiply (or reduce) an inequality by a variable (or the expression with a variable) if you don't know its sign.

Other tips on Inequalities are here: https://gmatclub.com/forum/inequalities- ... 75001.html



Hi bunuel,
Can we not simply flip the inequality sign by taking reciprocal on both sides since both sides have odd powers (ie both shall have same signs) ?
so we'll get x^5 < x^3
Is this logic correct?


You are basically multiplying 1/x^5 > 1/x^3 by x^3 and then by x^5. If x > 0, then you'd keep the sign both times and get x^3 > x^5. If < 0, then you'd flip the sign twice and still get x^3 > x^5. So, yes 1/x^5 > 1/x^3 is equivalent to x^3 > x^5: both are true when x < -1 and 0 < x< 1.


You could also multiply 1/x^5 > 1/x^3 by x^6, which would be positive, and get x > x^3, which is also equivalent to 1/x^5 > 1/x^3 and true when x < -1 and 0 < x < 1.

Hope it helps.
Manager
Manager
Joined: 11 Dec 2020
Posts: 135
Own Kudos [?]: 46 [0]
Given Kudos: 55
Location: United States (NY)
Concentration: Operations, Economics
Schools: Booth '26
GPA: 3.7
Send PM
Re: D01-10 [#permalink]
Bunuel wrote:
Official Solution:


Is \(x\) greater than 1?

(1) \(\frac{1}{x} \gt - 1\).

Multiply \(\frac{1}{x} \gt - 1\) by \(x^2\) (we can safely do that because a nonzero number in even power is positive) to get \(x > -x^2\);

\(x +x^2 > 0\);

\(x(1 + x) > 0\).

Two cases:

a. Both multiples are positive, (\(x > 0\) and \(x + 1 > 0\)), so \(x > 0\);

b. Both multiples are negative, (\(x < 0\) and \(x + 1 < 0\)), so \(x < -1\);

So, \(\frac{1}{x} \gt - 1\) is true when \(x < -1\) or when \(x > 0\). Thus \(x\) may or may not be greater than 1. Not sufficient.

(2) \(\frac{1}{x^5} > \frac{1}{x^3}\).

Multiply \(\frac{1}{x^5} > \frac{1}{x^3}\) by \(x^6\) (we can safely do that because a nonzero number in even power is positive) to get \(x > x^3\)

\(x - x^3 > 0\)

\(x(1 - x^2) > 0\)

Two cases:

a. Both multiples are positive, (\(x > 0\) and \(1 - x^2 > 0\)), so \(0 < x < 1\). (Here is why: \(1 - x^2 > 0\) is the same as \(1 > x^2\), which gives \(-1 < x < 1\), which together with \(x > 0\) gives \(0 < x < 1\));

b. Both multiples are negative, (\(x < 0\) and \(1 - x^2 < 0\)), so \(x < -1\) (Here is why: \(1 - x^2 < 0\) is the same as \(1 < x^2\), which gives \(x < -1\) or \(x > 1\), which together with \(x < 0\) gives \(x < -1\)).

So, \(\frac{1}{x^5} > \frac{1}{x^3}\) is true when \(0 < x < 1\) or when \(x < -1\).

ANY \(x\) from these ranges will be less than 1. So, the answer to our original question (Is \(x\) greater than 1?), is NO. Sufficient.


Answer: B





Hi Bunuel,
we multiplied the satement 2 by x^6, but how do we know that x not zero?
Math Expert
Joined: 02 Sep 2009
Posts: 92913
Own Kudos [?]: 618949 [0]
Given Kudos: 81595
Send PM
Re: D01-10 [#permalink]
Expert Reply
Stanindaw wrote:
Bunuel wrote:
Official Solution:


Is \(x\) greater than 1?

(1) \(\frac{1}{x} \gt - 1\).

Multiply \(\frac{1}{x} \gt - 1\) by \(x^2\) (we can safely do that because a nonzero number in even power is positive) to get \(x > -x^2\);

\(x +x^2 > 0\);

\(x(1 + x) > 0\).

Two cases:

a. Both multiples are positive, (\(x > 0\) and \(x + 1 > 0\)), so \(x > 0\);

b. Both multiples are negative, (\(x < 0\) and \(x + 1 < 0\)), so \(x < -1\);

So, \(\frac{1}{x} \gt - 1\) is true when \(x < -1\) or when \(x > 0\). Thus \(x\) may or may not be greater than 1. Not sufficient.

(2) \(\frac{1}{x^5} > \frac{1}{x^3}\).

Multiply \(\frac{1}{x^5} > \frac{1}{x^3}\) by \(x^6\) (we can safely do that because a nonzero number in even power is positive) to get \(x > x^3\)

\(x - x^3 > 0\)

\(x(1 - x^2) > 0\)

Two cases:

a. Both multiples are positive, (\(x > 0\) and \(1 - x^2 > 0\)), so \(0 < x < 1\). (Here is why: \(1 - x^2 > 0\) is the same as \(1 > x^2\), which gives \(-1 < x < 1\), which together with \(x > 0\) gives \(0 < x < 1\));

b. Both multiples are negative, (\(x < 0\) and \(1 - x^2 < 0\)), so \(x < -1\) (Here is why: \(1 - x^2 < 0\) is the same as \(1 < x^2\), which gives \(x < -1\) or \(x > 1\), which together with \(x < 0\) gives \(x < -1\)).

So, \(\frac{1}{x^5} > \frac{1}{x^3}\) is true when \(0 < x < 1\) or when \(x < -1\).

ANY \(x\) from these ranges will be less than 1. So, the answer to our original question (Is \(x\) greater than 1?), is NO. Sufficient.


Answer: B





Hi Bunuel,
we multiplied the satement 2 by x^6, but how do we know that x not zero?


We are given that \(\frac{1}{x^5} > \frac{1}{x^3}\). If x were 0, this statement would not be true because division by 0 is not allowed.
Math Expert
Joined: 02 Sep 2009
Posts: 92913
Own Kudos [?]: 618949 [0]
Given Kudos: 81595
Send PM
Re: D01-10 [#permalink]
Expert Reply
I have edited the question and the solution by adding more details to enhance its clarity. I hope it is now easier to understand.
DI Forum Moderator
Joined: 05 May 2019
Status:GMAT Club Team member
Affiliations: GMAT Club
Posts: 1031
Own Kudos [?]: 638 [0]
Given Kudos: 1003
Location: India
GMAT Focus 1:
645 Q82 V81 DI82
GMAT 1: 430 Q31 V19
GMAT 2: 570 Q44 V25
GMAT 3: 660 Q48 V33
GPA: 3.26
WE:Engineering (Manufacturing)
Send PM
Re D01-10 [#permalink]
I think this is a high-quality question and I agree with explanation.
Intern
Intern
Joined: 11 May 2023
Posts: 38
Own Kudos [?]: 12 [0]
Given Kudos: 25
Location: India
Concentration: Technology, General Management
GMAT 1: 670 Q47 V35
GMAT 2: 650 Q47 V35
Send PM
Re: D01-10 [#permalink]
Quote:
Bunuel
are there any questions like this that we can solve? I think the trick of multiplying by an even power of x is really useful.
Math Expert
Joined: 02 Sep 2009
Posts: 92913
Own Kudos [?]: 618949 [0]
Given Kudos: 81595
Send PM
Re: D01-10 [#permalink]
Expert Reply
aasdf1234 wrote:
Quote:
Bunuel
are there any questions like this that we can solve? I think the trick of multiplying by an even power of x is really useful.


You can find a bunch of questions where you can apply the same technique in our DS Inequality database.
Intern
Intern
Joined: 24 Jul 2023
Posts: 8
Own Kudos [?]: 3 [0]
Given Kudos: 305
Send PM
Re: D01-10 [#permalink]
I took such an easy route. St. 1 gives us nothing to answer x>1?
But St.2 very clear... if x>1 then x^5 should be > than x^3... and with high denominator the reciprocal of x^5 should be smaller, but that is not the case. Therefore it is suffiicient to say that whatever the case maybe, x is definitely NOT > than 1.
GMAT Club Bot
Re: D01-10 [#permalink]
Moderator:
Math Expert
92912 posts

Powered by phpBB © phpBB Group | Emoji artwork provided by EmojiOne